These triangles are scaled copies of each other. Four right triangles labeled F, B, G, and H. Triangle F has the vertical side labeled 8, the horizontal side labeled 6, and the longest side labeled 10. Triangle B has the vertical side labeled 2, the horizontal side labeled 3 halves, and the longest side labeled 5 halves. Triangle G has the vertical side labeled 4, the horizontal side labeled 3, and the longest side labeled 5. Triangle H has the vertical side labeled 8 thirds, the horizontal side labeled 2, and the longest side labeled 10 thirds. For each pair of triangles listed, the area of the second triangle is how many times larger than the area of the first? Remember that area is scale factor x scale factor. Use Triangle G and Triangle F

Answers

Answer 1

Answer:

The question is not complete, below is the completed question:

These triangles are scaled copies of each other. Four right triangles labeled F, B, G, and H. Triangle F has the vertical side labeled 8, the horizontal side labeled 6, and the longest side labeled 10. Triangle B has the vertical side labeled 2, the horizontal side labeled 3 halves, and the longest side labeled 5 halves. Triangle G has the vertical side labeled 4, the horizontal side labeled 3, and the longest side labeled 5. Triangle H has the vertical side labeled 8 thirds, the horizontal side labeled 2, and the longest side labeled 10 thirds. For each pair of triangles listed, the area of the second triangle is how many times larger than the area of the first? Remember that area is scale factor x scale factor.

1. Triangle G and Triangle F

2. Triangle G and Triangle B

3. Triangle B and Triangle F

4. Triangle F and Triangle H

5. Triangle G and Triangle H

6. Triangle H and Triangle B

Answer:

1. Triangle G and Triangle F  = 4 times

2. Triangle G and Triangle B  = 0.2 times

3. Triangle B and Triangle F  = 16 times

4. Triangle F and Triangle H  = 0.11 times

5. Triangle G and Triangle H  = 0.44 times

6. Triangle H and Triangle B = 0.56 times

Step-by-step explanation:

using the image of the triangles attached to this solution, let us calculate the area of each pair of triangles

1. Area of Triangle = [tex]\frac{base\ \times\ height}{2}[/tex]

[tex]F = \frac{6 \times 8}{2} = \frac{48}{2} = 24\\B = \frac{\frac{3}{2} \times 2 }{2} = \frac{3}{2} = \frac{3}{2}\\ G = \frac{3 \times4}{2}= \frac{12}{2} = 6\\ H = \frac{2 \times \frac{8}{3} }{2} = \frac{16}{6} = \frac{8}{3}[/tex]

To find by how many times the second triangle is larger than the first, we will calculate the ratio of the areas of the second triangle to the first

1. Triangle G and Triangle F  = 24:6 = 4

∴ The area of triangle F is 4 times larger than triangle G

2. Triangle G and Triangle B  = 3/2 : 6 = [tex]\frac{1}{4}[/tex] = 0.2

∴ The area of triangle B is 0.2 times larger than triangle G

3. Triangle B and Triangle F  = [tex]24 : \frac{3}{2} = 16[/tex]

∴ The area of triangle F is 16 times larger than triangle B

4. Triangle F and Triangle H  = [tex]\frac{8}{3} : 24 = \frac{1}{9} = 0.11[/tex]

∴ The area of triangle H is 0.11 times larger than triangle F

5. Triangle G and Triangle H = [tex]\frac{8}{3} : 6 = \frac{4}{9} = 0.44[/tex]

∴ The area of triangle H is 0.44 times larger than triangle G

6. Triangle H and Triangle B = [tex]\frac{3}{2} : \frac{8}{3} = \frac{9}{16} = 0.56[/tex]

∴ The area of triangle B is 0.56 times larger than triangle H

These Triangles Are Scaled Copies Of Each Other. Four Right Triangles Labeled F, B, G, And H. Triangle

Related Questions

I need help please thanks
find x when y = 18, if y = 9 when x = 8
ASAP PLEASE

Answers

so

y=9 -> y=18

       *2

x=8 -> x=16

       *2

so x = 16

Answer:

the only logical way solving this question is:

if y=9 then x=8

if y=18 then x should be 16

Step-by-step explanation:

I cant find any other way to solve

Use algebraic means to show that x^2 + y^2 = 8 is not a function. Explain your process.

Answers

Answer:

For a single value of x function has more than one corresponding value of y which satisfies the equation.

Step-by-step explanation:

Function: A relationship between a set of inputs and a set of possible outputs, where exactly one output is associated with each input.

It means for an equation to represent a function any single value of  x there should be only one corresponding value of  y which satisfies the equation.

Now consider the given equation.

[tex]x^2 + y^2 = 8[/tex]

If we put x=0 then we get two value of y i.e [tex]\sqrt8[/tex] and [tex]-\sqrt8[/tex] which satisfy the equation  and therefore the equation is not a function.

Find the product: 5(2x2 + 4x + 3)

Answers

Answer:

10x^2+20x+15

Step-by-step explanation:

5(2x^2+4x+3)

u distributed it and u get

10x^2+20x+15

Answer: I am pretty sure it is 115+20x

Step-by-step explanation: You have to do distributive Property

10 times 10+ 20x+15

100+20x+15

115+20x

an airplane is change in alitude before landing is shown in the table

Answers

Answer:

wheres the table can you please provide one

Step-by-step explanation:

AnswerA= -4,000m

Step-by-step explanation:

1    35,000

2   31,000

3   27,000

4   23,000

Youre subtracting 4,000 everytime.

(HELP PLEASE) Lines a and d are


-non-coplanar.

-parallel.

-perpendicular.

-skew.

Answers

the lines are perpendicular

A dietician is considering three food types to be served at a meal she is particularly concerned with the amount of one vitamin provided?

5 years ago

at this meal ,. One ounce of food 1 provide 8 milligrams of the vitamin , an ounce of food 2 provides 24 milligram and an ounce of food 3 provides 16 milligrams . The minimum daily requirements for vitamin is 120 milligram

a. If x equals the number of ounces of food type j served at meal, determine the equation which ensure the meal satisfies MDR exactly.


i want linear equation solution of this

Answers

Answer:

8* x₁ + 24*x₂ + 16*x₃ ≥ 120

Step-by-step explanation:

food 1    provides    8 mgrs  of vitamin

food 2   provides 24  mgrs  of  vitamin

food 3   provides  16  mgrs of vitamin

If xj ( j = 1 ,2,3 ) is the quantity in ounces of food ( 1, 2, 3 ) then:

8* x₁ + 24*x₂ + 16*x₃ ≥ 120

hurry i'm doing a test so
what is the value of x in the equation 5.5x = 17.6?

Answers

Answer:

x=3.2

Step-by-step explanation:

Answer:

x=3.2

Step-by-step explanation:

Divide each term by  5.5  and simplify.

x = 3.2

Find the slope of the line

Answers

(0,5) is the answer

What integer represents the situation ""6 yards behind the winner""?

Answers

Answer:

-6

Step-by-step explanation:

An integer is a whole number that can either be positive, negative or zero.

Now, we want to write the integer that represents 6 yards behind the winner.

Now, a distance of 6 yards behind the winner means a deficit distance of 6 yards behind the winner. Which in turn means a negative distance which is -6 yards behind the winner.

Thus, the integer is -6

The population of Philadelphia, Pennsylvania, is about 1.5 × 106 people. The population of Moscow, Russia, is about 8 times greater than the population of Philadelphia. Drag the buttons to the correct places to show how to use a calculator to find the population of Moscow.

Answers

Answer:

8 x (1.5EE6)

Step-by-step explanation:

Which statement describes the vertical asymptotes of the graph of f(x) = x2–64/8x - 64

Answers

Answer:

The graph has no vertical asymptote.  or A

Step-by-step explanation:

Right on edge

For the given function f(x) = (x² -64) / (8x -64)  there is no vertical asymptotes of the graph.

What is vertical asymptotes ?

"A vertical asymptote is a vertical line, , which represents  that invisible line approaches from either the positive or negative side, the function approaches infinity. Vertical asymptotes occur at the values where a rational function has a denominator of 0."

According to the question,

Given,

f(x) = (x² -64) / (8x -64)

     = ( x- 8)(x + 8) / 8(x -8)

     = (x + 8) / 8

    = x/8 + 1

To find vertical asymptotes denominator = 0.

But here denominator 8 ≠ 0

Therefore,

No vertical asymptotes

horizontal asymptotes: no

slant/ oblique asymptotes: y = x/8 + 1

Hence, the graph has no vertical asymptotes.

Learn more about vertical asymptotes here

https://brainly.com/question/11743529

#SPJ2

4(3x + 1)
What is the answer to 4(3x + 1)

Answers

12x + 4

Divide 12 by 4

X=3

Answer:

Answer is 12x+4. Hope that helped.

You can buy a pack of 12 batteries
for $6. What is the unit price?

Answers

Answer:

Each battery costs .5 dollars or 50 cents

Step-by-step explanation:

6/12=.5

Answer:

total units 12

cost per unit $ 0.5000

Step-by-step explanation:

Hope this helps! :)

890 g of copper is mixed with 100 g of zinc to form brass. The density of copper is 8.9 g/cm³ and the density of zinc is 7.1 g / cm ³ .

a ) What is the volume of copper and zinc used to make the brass ?
b ) Calculate the density of the brass . ​

Answers

Density ρ is equal to mass m per unit volume v, or

ρ = m / v

Solve for volume:

v = m / ρ

(a) 890 g of copper, which has a density of 8.9 g/cm³, has a volume of

v = (890 g) / (8.9 g/cm³) = 100 cm³

100 g of zinc, with density 7.1 g/cm³, has a volume of

v = (100 g) / (7.1 g/cm³) ≈ 14.08 cm³

(b) The density of brass is then

ρ = (890 g + 100 g) / (100 cm³ + 14.08 cm³) ≈ 8.68 g/cm³

Maureen reads at an average rate of 18 pages per hour. If she reads for 3.5 hours how many pages will she read?

Answers

the answer is 63

Step-by-step explanation

18 x 3.5=63

Answer: 63 pages

Explanation: 18 times 3 equals 54 and then for half an hour that's 9 minutes added to 54 which is 63 pages.

8/9 - 1/3 equals what​

Answers

Answer: 5/9

Explanation: Start by making them have common denominators by multiplying the numerator and denominator by 3, so now its 8/9-3/9 then 8-3=5 so the answer is 5/9.
tha answer is excalty 5/9

Solve x2 - 4x + 1 = 0.
x= 2 + 2i15
x = 4+ V3
x = 2/3
x= -2-3

Answers

Answer:

x = 2 ± √ 3

Step-by-step explanation:

The product of two integers is -40. If
the sum of the two integers is -3, what
are the two integers?

Answers

Answer:

-20

Step-by-step explanation:

no cap

Aloma, a university graduate who started a successful business, wants to start an endowment in her name that will provide scholarships to ChE students. She wants the scholarship to provide $13,000 per year and expects the first one to be awarded on the day she fulfills the endowment obligation. If Aloma plans to donate $240,000, what rate of return must the university realize in order to award the annual scholarship forever?


The rate of return that the university must realize in order to award the annual scholarship forever is _____%.

Answers

Answer:

5.73%

Step-by-step explanation:

Calculation for the rate of return that the university must realize

Using this formula

Rate of return = Initial payment/(Plan to donate- Initial payment)

Let plug in the formula

Rate of return=$13,000/($240,000-$13,000)

Rate of return=$13,000/$227,000

Rate of return=0.0573*100

Rate of return =5.73%

Therefore The rate of return that the university must realize in order to award the annual scholarship forever is 5.73 %.

Un abuelo quiere repartir una cierta cantidad de manera proporcional entre sus tres nietos de 14, 10 y 6 años de edad ¿A quién le corresponde más? Resuelve el problema apoyándote con los videos previos. Una vez resuelto, planteen el tema a alguien de su familia y juntos comenten cómo utilizan las relaciones proporcionales en sus quehaceres.

Answers

Answer:

El nieto de 14 años de edad recibirá la mayor cantidad.

Step-by-step explanation:

En este problema, debemos tener en mente que la cantidad aportada es directamente proporcional a la edad de quien recibe, esta cantidad es igual al producto de la cantidad total y la razón de la edad de quien recibe y la suma de todas las edades.

Bajo este razonamiento, podemos concluir que el nieto de 14 años de edad recibirá la mayor cantidad.

HELP : calculating slope between two points.

Answers

Answer:

(1.)3.5 (2.)-3 (3)0

Step-by-step explanation:

if right comment please

Your answers are
1. m=7/2
2.m=-3
3.m=0

Look at the pattern below.

0, 3, 6, 9, ...

Create a new pattern that follows the same rule as the one above.

Answers

0 , 4 , 8 , 12 , .... that’s what you can do to answer that question

0,5,10,15,20,

Step-by-step explanation:

I'm just adding 5 to each because I add 3 to each number

Winter coats are marked 70% off on February 1st. if a coat costs $57.75 on January 31st, how much will it cost on February 1st.

Answers

Answer:

57.75-(57.75*0.7)=17,325

Or 100-70=30

So 57.75*0.3=17.325

Answer:

$40.425

Step-by-step explanation:

hope it helps good luck

Hanna lost 4 points on a test and earned 4 points on an extra-credit question​

Answers

Answer:

the total points earned and lose cancel eachother out to be 0. she did not lose any points

Answer:

the total points earned and lose cancel eachother out to be 0. she did not lose any points

Step-by-step explanation:

idk the answer at all​

Answers

Answer:

y-1=3/2(x+3)

Step-by-step explanation:

Calculate the slope m of the given line using the slope formula

m = (y₂ - y₁ ) / (x₂ - x₁ )

with (x₁, y₁ ) = (- 2, - 4) and (x₂, y₂ ) = (2, 2) ← 2 points on the line

m=2+4/2+2=6/4=3/2

Parallel lines have equal slopes.

The equation of a line in point-slope form is

y - b = m(x - a)

where m is the slope and (a, b) a point on the line

Here m = 3/2 and (a, b) = (- 3, 1), thus

y - 1 = 3/2 (x - (- 3)), that is

y - 1 = 3/2 (x + 3) ← equation in point- slope form

the info about it is down below enjoy your day :)

Answers

Step-by-step explanation:

column 2 is greater

have a gret day too

B is the correct answer. The number in column ll is greater. The decimal value of the number in the 2nd column is 0.78 and the value of the number in the first column is 0.52. Thence b is right;)

What is the midpoint of the segments with endpoints (3,7) and (9,15)

Answers

The midpoint is (6, 11)

I have attached the equation so it’s easier to understand.

To find the midpoint we do
(x^1+x^2 /2 , y^1+y^2 /2)

3, 7 is x^1, y^1
And
9, 15 is x^2, y^2

(3+9 /2 , 7+15 /2)

Add the x and y =

(12/2 , 22/2)

Now we divide both by 2 =

(6, 11)

Answer:

(6,11)

I can confirm that this question is right.

12/2      22/2

(6   ,        11)

Sequence: 49, 36, 25, 16, 8, 4, 1 * Is this a arithmetic or geometric sequence? and if it is arithmetic find the common difference and if it is geometric find the common ratio? and please do not give me a stupid answer only smart people answer this if you know the answer to this you are smart. 1

Answers

Answer:

The given sequence [tex]49,36,25,16,8,4,1[/tex] is neither a arithmetic nor a geometric sequence

Step-by-step explanation:

A sequence is said to be an arithmetic sequence if difference between the terms is same.

A sequence is said to be a geometric sequence if ratio of the terms is same.

Given sequence is [tex]49,36,25,16,8,4,1[/tex]

Here,

[tex]36-49=-13\\25-36=-11\\36-49\neq 25-36[/tex]

So, this sequence is not an arithmetic sequence.

Also,

[tex]\frac{36}{49}\neq \frac{25}{36}[/tex]

So, this sequence is not a geometric sequence.

Therefore,

The given sequence [tex]49,36,25,16,8,4,1[/tex] is neither a arithmetic nor a geometric sequence

Analyzing a System
Solve the system of inequalities.
x² - gpg
v> 2
Which describe the solution set of the given system?
Check all that apply
The solution set is unbounded.
The point (1, 0) is in the solution set.
There is one point of intersection of the boundary
curves
The boundaries of the overlapping region are in the
solution set.
The boundary curves intersect at approximately
(-3,0).

Answers

Answer:

A, C, E

Step-by-step explanation:

took the assignment on edge

Answer:

a , c and e

Step-by-step explanation:

got it right on edg ^u^

A candy company claims that 20% of the candies in its bags are colored green. Steve buys 30 bags of 30 candies, randomly selects one candy from each, and counts the number of green candies. If there are 5, 6, or 7 green candies, Steve will conclude that the company’s claim is correct. What is the probability of Steve agreeing with the company’s claim? Use a TI-83, TI-83 plus, or TI-84 calculator to find the probability.

Answers

Answer:

The probability of Steve agreeing with the company’s claim is 0.50502.

Step-by-step explanation:

Let X denote the number of green candies.

The probability of green candies is, p = 0.20.

Steve buys 30 bags of 30 candies, randomly selects one candy from each, and counts the number of green candies.

So, n = 30 candies are randomly selected.

All the candies are independent of each other.

The random variable X follows a binomial distribution with parameter n = 30 and p = 0.20.

It is provided that if there are 5, 6, or 7 green candies, Steve will conclude that the company’s claim is correct.

Compute the probability of 5, 6 and 7 green candies as follows:

[tex]P(X=5)={30\choose 5}(0.20)^{5}(1-0.20)^{30-5}=0.17228\\\\P(X=6)={30\choose 6}(0.20)^{6}(1-0.20)^{30-6}=0.17946\\\\P(X=7)={30\choose 7}(0.20)^{7}(1-0.20)^{30-7}=0.15328[/tex]

Then the probability of Steve agreeing with the company’s claim is:

P (Accepting the claim) = P (X = 5) + P (X = 6) + P (X = 7)

                                       = 0.17228 + 0.17946 + 0.15328

                                       = 0.50502

Thus, the probability of Steve agreeing with the company’s claim is 0.50502.

Other Questions
The higher something is off the ground, the morepotential energy it has.TrueFalse a positive real number is 2 more than another. if the sum of the squares of the two numbers is 14, find the numbers A body moves with a velocity of 30.0 m/s.This speed is increased to 40 m/s in 4.0s.Calculatea) the average acceleration of the body.b) the distance covered by the body. Drag the numbers to the correct locations on the image. Not all numbers will be used. Complete the synthetic division to find the quotient of 3x3 - 25x2 + 12x - 32 and x - 8 1. Which level of biological organization, from smallest to largest, iscorrect? *A. Organ System, Organ, Tissue, CellB. Cell, Organ, Tissue, Organ SystemC. Cell, Tissue, Organ, Organ SystemD. Tissue, Cell, Organ, Organ System What is the value of x in the equation 8+4 = 2(x-1)?5112132.7 How would you describe the status of the Seven Commandments of the Principles of Animalism at the end of the stor What does the sodium-potassium pump keep at a high concentration outside of the cell?A. potassiumB. sodium I would like to get some answers for my test but it's not letting me see the answers For your biology class, you have taken a number of measurements for a plant growth experiment. You wish to create a chart that shows what the progress looks like over time. Which application is best suited for this effort?Notepad or PaintImpress or PowerPointWriter or WordCalc or Excel A book shop offers a 24% discount on books.Sam has to pay $______???? if he purchases books with an original price of $84. -4(3+7m) please help i will give brainlest and do not answer just to get points this is DUE IN FIVE MINUTES Who'd a thunk it, Mabel, Why does Grim say this to Gram on page 46?What is he referring to ? Please explain. *4 pointsWhod a thunk it, Mabel why does grim say that to gram on page 46 The last day of school is always an amazing day for myfriends and me. First, we eat at our favorite pizzarestaurant. Then, after we've stuffed ourselves, we go tothe bowling alley. After bowling a couple of games, wehead to the movies for the latest horror movie. If we stillhave time left, we head back to the pizza place for morefood.A. for my friends and meB. If we still have time leftC. we go to the bowling alleyD. for the latest horror movieI NEED IT ASAP APEX!!! match the vocabulary 1 The Emergence of Empires Attempt of i SECTION 10F 1 5SIGNMENTS COURSES Match the vocabulary term with its meaning a tax or payment in goods or 1. relief money a projection from the surface of an 2. empire art piece a political unit made up of a vast 3. tribute expanse of territory that is under one ruler a Greek statesman who ended the 4. Solon rule of the tyrants Greek lawgiver who initiated many 5. Cleisthenes reforms in Greek society George rented a bike to tour a nearby park. The charge to rent the bike is a flat rate of $10.75 plus $4.50 per each hour that he spends riding the bike. How much did George pay if he was riding for 3 1 fourth hours? -Maggie turned right out of the school and drove 5 miles East to the store. She left the store and drove 2 miles North to Pelican's, then drove 5 miles South to her friend's house. When it was time to pick up her brother, she drove 7 miles West, then drove 4 miles North. Maggie and her brother then drove 2 miles East to get home. What is the displacement from Maggie's home to school? A student is working two jobs over the summer to pay for a new car. The student gets paid $25 for each lawn thatis mowed ( 2 )and $15 for each pool that is cleaned (y). The student wants to earn $750 to pay for a down paymenton a car.What is the y-intercept What is the resultant velocity of a plane that is traveling at 245 m/s North and encounters a tailwind of 55 m/s North? (MC)The drawing below shows a scene from the South during Reconstruction. The illustration shows a line of African American men and women at the window of a building. The window is open and a white man sits in the window. An African American woman hands a piece of paper to the white man. Near the woman, a man and a child stand with the man holding a basket. Library of Congress [LC-USZ62-37861] What activity does this drawing depict? African Americans joining the Union Army a Republican postmaster registering new voters the Freedmen's Bureau distributing supplies slaves buying railroad tickets to flee to the North